You are on page 1of 1

Management Control System

MBA Sem III Preliminary Examination/PGPBA Sem III Final Examination


Date: 28th October 2005 : 9.30 to 11.30 am
Marks: 40 Time: 2 Hours
Question 1 is compulsory & Solve any 3 from rest; All questions carry equal marks.
===========================================================
Que.1 : M/s Super Apparels Ltd. is engaged in manufacture of kids’ ware. The annual
cost and sales data for the company is given as under

Sales (9000 units @ Rs 100) Rs. 9,00,000


Cost of Sales 6,40,000
Gross Profit 2,60,000
Selling & Administrative Expenses 1,50,000
Profit 1,10,000

The installed capacity of the company is 10,000 units per year. The cost of goods sold
includes the fixed cost of 1,00,000. The variable sales and administration expenses are:
4% sales commission and Rs. 2 per unit paid to the designer. A departmental store
manager has approached the sales manager of the company i.e. M/s Super Apparels Ltd,
with an offer to purchase 1,000 units at Rs. 70 per unit. The said order will not attract any
sales commission payment and designer has also agreed to reduce his fees by 50% on any
number of units sold to this store.
a) Offer your advice to the sales manager of the M/s Super Apparels Ltd
b) Suppose the order would be for 2,500 units instead of 1,000; would it have made any
difference to your decision?
c) Assuming the same facts as in (b) above, what is the lowest price that the firm would
accept and still earn Rs. 1,10,000?

Que 2. : “Balanced scorecard tells the story of your company’s strategy”


Support the statement with the help of examples

Que 3 : What are the advantages of Economic Value Added over Return on Investment?
Describe significance of each with the help of an example.

Que 4 : “When it is quite certain that the basic objective of any business activity is to
earn money, then why a management controller should worry about non-
financial performance measures?” Discuss with the help of an example.

Que 5 : When every department and its head have to strive to achieve its own goal, does
it not affect goal congruence process? Explain with the help of an
example.

Que 6 : Marketing manager of the company proposes to enhance the market penetration
by employing more than proportional resources. Being a management
controller of the company do you support his proposal, if so specify and
discuss the considerations involved and suggest a course of action for him.
Make necessary assumptions.

You might also like